LSAT and Law School Admissions Forum

Get expert LSAT preparation and law school admissions advice from PowerScore Test Preparation.

User avatar
 Dave Killoran
PowerScore Staff
  • PowerScore Staff
  • Posts: 5852
  • Joined: Mar 25, 2011
|
#72711
This game is also discussed in our Podcast, at the 24:24 mark: LSAT Podcast Episode 37: The November 2019 LSAT Logic Games Section


Complete Question Explanation

(The complete setup for this game can be found here: https://forum.powerscore.com/lsat/viewtopic.php?t=31745)

The correct answer choice is (C).

Answer choice (A): This answer is incorrect because from the third rule either P or Y must be included.

Answer choice (B): This answer is incorrect because from the last rule O must be added before G.

Answer choice (C): This is the correct answer.

Answer choice (D): This answer is incorrect because from the second rule J and Y cannot be added together.

Answer choice (E): This answer is incorrect because from the first rule when J is added it must be first or last.

Get the most out of your LSAT Prep Plus subscription.

Analyze and track your performance with our Testing and Analytics Package.